6

How can i evaluate $$\int _0^{\frac{\pi }{2}}x\ln \left(\sin \left(x\right)\right)\:dx$$ I started like this $$\int _0^{\frac{\pi }{2}}x\ln \left(\sin \left(x\right)\right)\:dx=\frac{x^2\ln \left(\sin \left(x\right)\right)}{2}|^{\frac{\pi }{2}}_0-\frac{1}{2}\int _0^{\frac{\pi }{2}}x^2\cot \left(x\right)\:dx$$ but this way doesnt turn things any simpler, i also tried using the substitution $t=\tan \left(\frac{x}{2}\right)$ and got this, $$4\int _0^{1}\arctan \left(t\right)\ln \left(\frac{2t}{1+t^2}\right)\:\frac{1}{1+t^2}\:dt$$ $$=4\ln \left(2\right)\int _0^{1}\frac{\arctan \left(t\right)}{1+t^2}\:dt+4\int _0^{1}\frac{\arctan \left(t\right)\ln \left(t\right)}{1+t^2}\:dt-4\int _0^{1}\frac{\arctan \left(t\right)\ln \left(1+t^2\right)}{1+t^2}\:dt$$ That first integral is very simple but the rest look very difficult, could you help me evaluate this one?

  • Is the integral converging (at 0)? – markvs Jul 08 '20 at 05:50
  • According to Wolfram Alpha, the answer involves $\zeta(3)$, So it is not quite an easy thing to find the integral exactly. – markvs Jul 08 '20 at 05:55
  • 1
    Use the fourier series of ln(sinx) and I think this integral is possible duplicate. – Ali Olaikhan Jul 08 '20 at 05:56
  • The $x$ complicates matters. Without it, $\displaystyle \int_0^{\frac{\pi}{2}} \ln(\sin x) dx$ is easy to find. – Deepak Jul 08 '20 at 06:06
  • @астонвіллатересалисбон no need to be so unkind. –  Jul 08 '20 at 06:07
  • 1
    @maurijim I saw nothing unkind in what he wrote. Wolfram Alpha should always be the first place you check if an integral looks difficult. And you haven't given much context for this problem - where did it come from? – Deepak Jul 08 '20 at 06:12
  • @Deepak no i get that and i agree but that last part what just uncalled for. Me and my classmates came up with this integral after solving a similar case. –  Jul 08 '20 at 06:21
  • @maurijim I apologize, it was not my intention. The point was, after insertion into Wolfram Alpha I spotted $\zeta(3)$, the Apery constant which doesn't have easy integral representations. I guessed that you will not know this , and therefore reported that the question may be out of reach. As the answer below shows, it is not easy, but I was rash in stating this. Sorry once again. You can ask for an explanation if you don't understand some of the steps below. – Sarvesh Ravichandran Iyer Jul 08 '20 at 06:47
  • See https://math.stackexchange.com/questions/3745615/evaluate-int-01-arctan3-x-dx – FDP Jul 08 '20 at 19:09

7 Answers7

9

$$\int_0^{\pi/2}x\ln(\sin x)dx=\int_0^{\pi/2}x\left(-\ln2-\sum_{n=1}^\infty\frac{\cos(2nx)}{n}\right)dx$$

$$=-\frac{\pi^2}{8}\ln2-\sum_{n=1}^\infty\frac{1}{n}\int_0^{\pi/2}x\cos(2nx)dx$$

$$=-\frac{\pi^2}{8}\ln2-\sum_{n=1}^\infty\frac{1}{n}\left(\frac{\cos(n\pi)}{4n^2}+\frac{\pi\sin(n\pi)}{4n}-\frac{1}{4n^2}\right)$$

$$=-\frac{\pi^2}{8}\ln2-\sum_{n=1}^\infty\frac{1}{n}\left(\frac{(-1)^n}{4n^2}+\frac{0}{4n}-\frac{1}{4n^2}\right)$$

$$=-\frac{\pi^2}{8}\ln2-\frac14\text{Li}_3(-1)+\frac14\zeta(3)$$

$$=-\frac{\pi^2}{8}\ln2+\frac{7}{16}\zeta(3)$$


Bonus: With subbing $x\to \pi/2-x$ we have

$$\int_0^{\pi/2}x\ln(\cos x)dx=\int_0^{\pi/2}(\pi/2-x)\ln(\sin x)dx$$

$$=\frac{\pi}{2}\int_0^{\pi/2}\ln(\sin x)dx-\int_0^{\pi/2}x\ln(\sin x)dx$$

$$=\frac{\pi}{2}\left(-\frac{\pi}{2}\ln2\right)-\left(-\frac{\pi^2}{8}\ln2+\frac{7}{16}\zeta(3)\right)$$ $$=-\frac{\pi^2}{8}\ln(2)-\frac7{16}\zeta(3)$$

Or we can use the Fourier series of $\ \ln(\cos x)=-\ln2-\sum_{n=1}^\infty\frac{(-1)^n\cos(2nx)}{n}$.

Also by subtracting the two integrals gives

$$\int_0^{\pi/2}x\ln(\tan x)dx=\frac78\zeta(3)$$

Or we can use the Fourier series of $\ \ln(\tan x)=-2\sum_{n=1}^\infty\frac{\cos((4n-2)x)}{2n-1}.$

Ali Olaikhan
  • 27,891
  • Thanks! the solution was neat and easier than expected though i have one question ¿what were the conditions that were met that made the usage of the fourier series possible/valid? –  Jul 08 '20 at 07:00
  • 1
    You welcome. It holds for x between zero and pi. – Ali Olaikhan Jul 08 '20 at 07:28
  • 1
    That last result is absolutely amazing, because I just see it beautifully capturing $\zeta(3)$ in integral form. This question has been nice to work on, +1 for both q and a. – Sarvesh Ravichandran Iyer Jul 08 '20 at 16:37
4

Define on $[0;\infty[$ the function $R$ by,

for all $x\in [0;\infty[$, $\displaystyle \text{R}(x)=\int_0^x \dfrac{\ln t}{1+t^2}\,dt=\int_0^1 \dfrac{x\ln(tx)}{1+t^2x^2}\,dt$.

Observe that $\text{R}(0)=\lim_{x\rightarrow}\text{R}(x)=0$ \begin{align} A_2&=\int_0^{\frac{\pi}{2}}t\ln(\cos t)\,dt\\ B_2&=\int_0^{\frac{\pi}{2}}t\ln(\sin t)\,dt\\ A_2+B_2&=\int_0^{\frac{\pi}{2}}t\ln\left(\frac{1}{2}\sin(2t)\right)\,dt\\ &=\int_0^{\frac{\pi}{2}}t\ln\left(\sin(2t)\right)\,dt-\frac{\pi^2\ln 2}{8}\\ &\overset{x=2t}=\frac{1}{4}\int_0^\pi x\ln(\sin x)\,dx-\frac{\pi^2\ln 2}{8}\\ &\overset{t=\pi-x}=\frac{1}{4}\int_0^\pi (\pi-x)\ln(\sin x)\,dx-\frac{\pi^2\ln 2}{8}\\ 2(A_2+B_2)&=\frac{\pi}{4}\int_0^\pi \ln(\sin x)\,dx-\frac{\pi^2\ln 2}{4}\\ A_2+B_2&=\frac{\pi}{8}\int_0^\pi \ln(\sin x)\,dx-\frac{\pi^2\ln 2}{8}\\ &=-\frac{\pi^2\ln 2}{4}\\ B2-A2&=\int_0^{\frac{\pi}{2}}t\ln(\tan t)\,dt\\ &\overset{x=\tan t}=\int_0^\infty \frac{\ln x\arctan x}{1+x^2}\,dx\\ \end{align} \begin{align} U_2&=\int_0^\infty \frac{\arctan\left(\frac{1}{x}\right)\ln x}{1+x^2}\,dx\\ V_2&=\int_0^\infty \frac{\arctan\left(x\right)\ln x}{1+x^2}\,dx\\ U_2+V_2&=\frac{\pi}{2}\int_0^\infty \frac{\ln x}{1+x^2}\,dx\\ &=0\\ U_2&\overset{\text{IBP}}=\left[R(x)\arctan\left(\frac{1}{x}\right)\right]_0^\infty +\int_0^\infty \frac{R(x)}{1+x^2}\,dx\\ &=\int_0^\infty \left(\int_0^1 \dfrac{x\ln(tx)}{(1+t^2x^2)(1+x^2)}\,dt\right)\,dx\\ &=\int_0^\infty \left(\int_0^1 \dfrac{x\ln(x)}{(1+t^2x^2)(1+x^2)}\,dt\right)\,dx+\\ &\int_0^1 \left(\int_0^\infty \dfrac{x\ln(t)}{(1+t^2x^2)(1+x^2)}\,dx\right)\,dt\\ &=V_2+\int_0^1 \frac{\ln^2 t}{t^2-1}\,dt\\ &=V_2+\int_0^1 \frac{t\ln^2 t}{1-t^2}\,dt-\int_0^1 \frac{\ln^2 x}{1-x}\,dx\\ &\overset{u=t^2}=B+\frac{1}{8}\int_0^1 \frac{\ln^2 t}{1-t}\,dt-\int_0^1 \frac{\ln^2 x}{1-x}\,dx\\ &=V_2-\frac{7}{8}\int_0^1 \frac{\ln^2 x}{1-x}\,dx\\ &=V_2-\frac{7}{8}\times 2\zeta(3)\\ &=V_2-\frac{7}{4}\zeta(3)\\ U_2&=-\frac{7}{8}\zeta(3)\\ V_2&=\frac{7}{8}\zeta(3)\\ B_2-A_2&=\frac{7}{8}\zeta(3)\\ A_2&=-\frac{7}{16}\zeta(3)-\frac{1}{8}\pi^2\ln 2\\ B_2&=\boxed{\frac{7}{16}\zeta(3)-\frac{1}{8}\pi^2\ln 2}\\ \end{align}

NB: I assume following results: \begin{align} \int_0^\infty \frac{\ln x}{1+x^2}\,dx&=0\\ \int_0^1 \frac{\ln^2 x}{1-x}\,dx&=2\zeta(3)\\ \int_0^\pi \ln(\sin x)\,dx&=-\pi\ln 2 \end{align}

FDP
  • 15,643
3

This solution is based on Cauchy's integral theorem.

Integrate \begin{equation*} f(z) = \log(z)\dfrac{\log(1-z^2)}{z} \end{equation*} where \begin{equation*} \log(z)=\ln|z|+i\arg(z), \qquad -\pi<\arg(z)<\pi, \end{equation*} over the boundary $\gamma$ of the unit circle in the first quadrant. Let $\gamma = \gamma_1+\gamma_2+\gamma_3$. Here \begin{alignat*}{1} \gamma_1(x)&=x,\, 0\le x\le 1\\ \gamma_2(t)&=e^{it}, \, 0 \le t \le {\pi}/2\\ \gamma_3(y)&=iy,\, y \mbox{ from } 1 \mbox{ to } 0. \end{alignat*} From Cauchy's integral theorem we get \begin{gather*} 0 = \int_{\gamma_1}f(z)\,\mathrm{d}z + \int_{\gamma_2}f(z)\,\mathrm{d}z + \int_{\gamma_3}f(z)\,\mathrm{d}z=\\[2ex] \int_{0}^{1}\ln(x)\dfrac{\log(1-x^2)}{x} \,\mathrm{d}x+ \int_{0}^{\pi/2}\log(e^{it})\dfrac{\log(1-e^{i2t})}{e^{it}}ie^{it} \,\mathrm{d}t- \int_{0}^{1}\log(iy)\dfrac{\log(1+y^2)}{iy}i\, \mathrm{d}y=\\[2ex] \int_{0}^{1}\ln(x)\dfrac{\log(1-x^2)}{x} \,\mathrm{d}x+ \int_{0}^{\pi/2}i^2t\left(\ln(2\sin(t))+i\arg\left(1-e^{i2t}\right)\right)\,\mathrm{d}t-\\[2ex] \int_{0}^{1}\left(\ln(y)+i\dfrac{\pi}{2}\right)\dfrac{\log(1+y^2)}{y}\, \mathrm{d}y. \end{gather*} We extract the real part of every integral. \begin{gather*} 0 = -\int_{0}^{1}\left(\sum_{k=1}^{\infty}\dfrac{x^{2k-1}\ln(x)}{k}\right)\, \mathrm{d}x - \int_{0}^{\pi/2}t\ln(2)\, \mathrm{d}t -\int_{0}^{\pi/2}t\ln(\sin(t))\, \mathrm{d}t-\\[2ex] -\int_{0}^{1}\left(\sum_{k=1}^{\infty}(-1)^{k-1}\dfrac{y^{2k-1}\ln(y)}{k}\right)\, \mathrm{d}y =\\[2ex] \sum_{k=1}^{\infty}\dfrac{1}{4k^3}-\dfrac{\pi^2}{8}\ln(2) -\int_{0}^{\pi/2}t\ln(\sin(t))\, \mathrm{d}t +\sum_{k=1}^{\infty}\dfrac{(-1)^{k-1}}{4k^3}=\\[2ex] \dfrac{1}{4}\zeta(3) -\dfrac{\pi^2}{8}\ln(2) -\int_{0}^{\pi/2}t\ln(\sin(t))\, \mathrm{d}t + \dfrac{3}{16}\zeta(3). \end{gather*} Consequently \begin{equation*} \int_{0}^{\pi/2}t\ln(\sin(t))\, \mathrm{d}t = \dfrac{7}{16}\zeta(3)-\dfrac{\pi^2}{8}\ln(2). \end{equation*}

JanG
  • 4,872
2

An incomplete solution requiring some more interesting work:

$$I=\int_{0}^{\pi/2} x \ln (\sin x) dx= \int_{0}^{1} \ln t ~\frac{\sin^{-1} t}{\sqrt{1-t^2}} dt.$$ Using the MacLaurin series for $$\frac{\sin^{-1} t}{\sqrt{1-t^2}}= \sum_{n=0}^{\infty} \frac{(2n)!!}{(2n+1)!!} t^{2n+1}$$

See: Deriving Maclaurin series for $\frac{\arcsin x}{\sqrt{1-x^2}}$. Then $$I=\sum_{n=0}^{\infty} \int_{0}^{1}\ln t ~\frac{(2n)!!}{(2n+1)!!} t^{2n+1}=-\sum_{n=0}^{\infty} \frac{(2n)!!}{(2n+1)!!}~ \int_{0}^{\infty}u~e^{-(2n+2)u}~du~~( t=e^{-u})$$ $$\implies I=-\sum_{n=0}^{\infty} \frac{(2n)!!}{(2n+1)!!} \frac{1}{(2n+2)^2}$$ I have numerically confirmed using Mathematica that $I$ is nothing but $$\frac{1}{16}[-\pi^2 \ln 4+7 \zeta(3)]$$ The same as obrained @Ali Shather in his very nice solution above. Can some one fill the gap here! I may come back.

Z Ahmed
  • 46,319
  • you can find some nice identities here https://de.wikibooks.org/wiki/Formelsammlung_Mathematik:_Reihenentwicklungen#Potenzen_des_Arkussinus that could be helpful. – Ali Olaikhan Jul 08 '20 at 21:33
1

Assuming that you could enjoy polylogarithms, the antiderivative does exist (have a look here) $$I=\int x\log \left(\sin \left(x\right)\right)\:dx$$ Using the bounds, the results are

  • at $\frac \pi 2$, $\frac{1}{48} \left(9 \zeta (3)+i \pi ^3-6 \pi ^2 \log (2)\right)$
  • at $0$, $\frac{1}{48} \left(i \pi ^3-12 \zeta (3) \right)$

and, then, the result.

0

The trapezoidal rule for numerical integration is given by: $$ \int_a^b f(x) \, dx \approx \frac{h}{2} \left[ f(a) + f(b) \right] + h \sum_{i=1}^{n-1} f(x_i) $$ where $h$ is the step size, $x_i = a + i h$ for $i = 1, 2, \dots, n-1$, and $h = \frac{b - a}{n}$.

For your integral from $1$ to $\frac{\pi}{2}$, you want the trapezoidal rule with just one subinterval (i.e., $n = 2$). This simplifies the formula to: $$ \int_1^{\frac{\pi}{2}} x \ln(\sin(x)) \, dx \approx \frac{h}{2} \left[ f(1) + f\left( \frac{\pi}{2} \right) \right] $$ where $h = \frac{\pi}{2} - 1$.

Apply the trapezoidal rule: $$ \int_1^{\frac{\pi}{2}} x \ln(\sin(x)) \, dx \approx \frac{1}{2} \cdot \left( \frac{\pi}{2} - 1 \right) \left[ f(1) + f\left( \frac{\pi}{2} \right) \right] $$

Evaluate $f(1)$ and $f\left( \frac{\pi}{2} \right)$: $$ f(1) = 1 \cdot \ln(\sin(1)) $$ $$ f\left( \frac{\pi}{2} \right) = \frac{\pi}{2} \cdot \ln(\sin\left( \frac{\pi}{2} \right)) = \frac{\pi}{2} \cdot \ln(1) = 0 $$

Thus, the integral approximation becomes: $$ \int_1^{\frac{\pi}{2}} x \ln(\sin(x)) \, dx \approx \frac{1}{2} \cdot \left( \frac{\pi}{2} - 1 \right) \left[ \ln(\sin(1)) + 0 \right] $$

Simplifying the expression: $$ \int_1^{\frac{\pi}{2}} x \ln(\sin(x)) \, dx \approx \frac{1}{2} \cdot \left( \frac{\pi}{2} - 1 \right) \ln(\sin(1)) $$

The trapezoidal rule approximation for the integral is: $$ \int_1^{\frac{\pi}{2}} x \ln(\sin(x)) \, dx \approx \frac{1}{2} \cdot \left( \frac{\pi}{2} - 1 \right) \ln(\sin(1)) $$ This is the final approximation using the trapezoidal rule with $n = 2$ subintervals.

Sebastiano
  • 8,290
0

Split the integrand into three as follows \begin{align} & \int _0^{\frac{\pi }{2}}x\ln \left(\sin x\right)\:dx\\ =&\ \int_0^\frac{\pi}{2}\frac x2\ln(\tan x)dx - \int_0^\frac{\pi}{2}x\ln2\ dx +\int_0^\frac{\pi}{2}\frac x2\ln(2\sin 2x)dx \\ = &\ \frac{7}{16}\zeta(3) -\frac{\pi^2}{8}\ln2+0 \end{align}

where $\int_{0}^{\frac{\pi}{2}}x\ln(\tan x)dx= \frac78\zeta(3)$ \begin{align} \int_0^\frac{\pi}{2} \hspace{-2mm} \underset{2x\to x} {x\ln(2\sin 2x)}dx =\int_0^{\pi} \frac x4\underset{x\to\pi-x} {\ln(2\sin x)}{dx} =&\frac\pi8 \int_0^{\pi}\ln(2\sin x)dx =0 \end{align}

Quanto
  • 120,125